Sull'inversione della trasformata di Laplace e su alcuni teoremi tauberiani. (Q2587166)

From MaRDI portal
scientific article
Language Label Description Also known as
English
Sull'inversione della trasformata di Laplace e su alcuni teoremi tauberiani.
scientific article

    Statements

    Sull'inversione della trasformata di Laplace e su alcuni teoremi tauberiani. (English)
    0 references
    1940
    0 references
    Vorankündigung einiger Resultate, deren Beweise in anderen Zeitschriften veröffentlicht werden sollen und von denen die folgenden angeführt seien. \(F (t)\) sei in jedem endlichen Intervall Lebesgue-integrabel, und \[ f (p) = \int\limits_{-\infty}^\infty e^{-pt} F (t)\, dt\qquad \text{(zweiseitige Laplace-Transformation)} \] sei für \(\alpha < \Re p < \beta\) konvergent. Für \(\lambda> 0\) und \(\alpha < k < \beta\) werde gesetzt: \[ G_{n,\lambda}(t)=\frac1{2\pi i}\int\limits_{k-i\lambda}^{k+i\lambda} e^{pt} f(p)\left(1+\frac{p^2}{\lambda^2}\right)^n\,dp. \] Dann ist für fast alle \(t\) \[ \lim_{\lambda\to\infty} G_{1,\lambda}(t) = F(t) \] (Verallgemeinerung der Riemannschen Umkehrformel). Wenn für ein festes \(t\) und ein bestimmtes ganzes \(n \geqq 0\) gilt: \(\lim\limits_{\lambda\to\infty}G_{n,\lambda}(t) = s\), so ist auch \(\lim\limits_{\lambda\to\infty} G_{n+1,\lambda}(t)=s\). In allen Stetigkeitspunkten von \(F\) ist für ganzes \(n>0\): \(\lim\limits_{\lambda\to\infty}G_{n,\lambda}(t) = F(t)\). Die folgenden Sätze beziehen sich auf die einseitige Laplace-Transformation \(f(p) = \int\limits_0^\infty e^{-pt}F(t)\,dt\). Vorausgesetzt wird: 1) Das Integral konvergiert für \(\Re p>\alpha\), und \(f(p)\) ist für \(\Re p> 0\) analytisch. 2) Für fast alle \(v\) existiert \(\lim\limits_{u\to+0} f(u + iv)\) als Funktion \(f (iv)\), die in jedem endlichen Intervall Lebesgue-integrabel ist. 3) Es existiert ein positives \(k> \alpha\), so daß für \(h> 0\) das Integral \(\int\limits_{-h}^v f(u + iv)\, dv\) im Intervall \(- h\leqq v \leqq h\) gleichgradig absolut stetig für \(0 < u \leqq k\) ist. 4) Es gibt eine Folge \(\lambda_1,\, \lambda_2,\,\ldots\) mit \(0 < \lambda_1 < \lambda_2 <\cdots \to \infty\), so daß \(\lim\limits_{u\to+0} f (u \pm i\lambda_n)\) existiert, und ein ganzes \(m > 0\), so daß \(\lim\limits_{n\to\infty} \dfrac{f(u\pm i\lambda_n)}{\lambda^m_n} = 0\) gleichmäßig hinsichtlich \(u\) für \(0\leqq u \leqq k\) gilt. -- Dann ist für fast alle \(t\) \[ F(t) = \lim_{n\to\infty} \frac1{2\pi} \int\limits_{-\lambda_n}^{\lambda_n} e^{ivt} f(iv) \left(1-\frac{v^2}{\lambda_n^2}\right)^m\, dv. \] Wenn \[ \lim_{t\to\infty} \int\limits_{-\lambda}^\lambda e^{ivt}f(iv) \left(1-\frac{v^2}{\lambda^2}\right)^m\, dv = 0 \] gleichmäßig für alle \(\lambda> 0\) ist, so gilt \(\lim\limits_{t\to\infty} F (t) = 0\). Der Schluß \(\lim\limits_{t\to\infty} F (t) = 0\) läßt sich auch auf Grund anderer Voraussetzungen ziehen, z. B. daß \(f (iv)\) von der Gestalt \(g(v)\psi(v)\) mit beschränktem \(g (v)\) und periodischem oder fastperiodischem \(\psi (v)\) ist, wobei allerdings bei dem Grenzübergang \(t \to\infty\) eine gewisse Folge von Intervallen vermieden werden muß. Eine weitere Gruppe von Sätzen beschäftigt sich mit dem Fall, daß \(f (iv)\) nicht integrabel ist, sondern Pole besitzt, wodurch die einander entsprechenden Darstellungen \(f(p) = \sum\dfrac{a_n}{p-i\lambda_n}\) und \(F (t) = \sum a_ne^{i\lambda_nt}\) auftreten.
    0 references
    0 references
    0 references